Diagonalization of a matrix with repeated eigenvalues

Click For Summary
Diagonalizing a matrix with repeated eigenvalues is possible, but requires understanding the multiplicity of eigenvalues and their corresponding eigenvectors. The identity matrix serves as a simple example, where the eigenvalue 1 has a multiplicity of 3. To find eigenvalues, one must solve the determinant equation det(A - λI) = 0, which in this case leads to (1 - λ)³ = 0, confirming 1 as a triple root. It's crucial to recognize that the equation does not yield unique solutions when eigenvalues are repeated. Understanding these concepts is essential for successful diagonalization.
bemigh
Messages
29
Reaction score
0
Hey guys,
I know its possible to diagonalize a matrix that has repeated eigenvalues, but how is it done? Do you simply just have two identical eigenvectors??
Cheers
Brent
 
Physics news on Phys.org
Well, try an example -- the simplest you can think of. How about the identity matrix? Can you diagonalize it? What are its eigenvectors?
 
Ok, well... i would get an eigenvalue of 1, with a multiplicity of 3. Alright, so when solving for my eigenvalues, by plugging 1 into my matrix A-(lamda)I ; i would just be left with 0's. How can i get eigenvalues from this?? I appreciate your help by the way...
 
Am I misunderstanding something? If you know what eigenvalues are, then you must know how to find them in this simple case. If \lambda is an eigenvalue of matrix A, then A-\lambda I is NOT invertible (since the equation A- \lambda I= 0 does not have a unique solution) and so det(A- \lambda I)= 0. Since you are finding the eigenvalues, you do NOT yet know that \lambda= 1 so you are NOT "left with 0's. In this case, that equation is (1- \lamba)^3= 0 which obviously has 1 as a triple root.
 
Question: A clock's minute hand has length 4 and its hour hand has length 3. What is the distance between the tips at the moment when it is increasing most rapidly?(Putnam Exam Question) Answer: Making assumption that both the hands moves at constant angular velocities, the answer is ## \sqrt{7} .## But don't you think this assumption is somewhat doubtful and wrong?

Similar threads

  • · Replies 8 ·
Replies
8
Views
2K
  • · Replies 7 ·
Replies
7
Views
2K
  • · Replies 3 ·
Replies
3
Views
2K
  • · Replies 5 ·
Replies
5
Views
2K
  • · Replies 2 ·
Replies
2
Views
1K
  • · Replies 4 ·
Replies
4
Views
1K
  • · Replies 4 ·
Replies
4
Views
1K
Replies
8
Views
3K
  • · Replies 2 ·
Replies
2
Views
2K
Replies
4
Views
2K